Respiratory practice questions

¡Supera tus tareas y exámenes ahora con Quizwiz!

A patient with chronic obstructive pulmonary disease (COPD) has poor gas exchange. Which action by the nurse would support the patient's ventilation? a. Have the patient rest in bed with the head elevated to 15 to 20 degrees. b. Encourage the patient to sit up at the bedside in a chair and lean forward. c. Ask the patient to rest in bed in a high-Fowler's position with the knees flexed. d. Place the patient in the Trendelenburg position with pillows behind the head.

Encourage the patient to sit up at the bedside in a chair and lean forward. Rationale: Patients with COPD improve the mechanics of breathing by sitting up in the "tripod" position. Resting in bed with the head elevated in a semi-Fowler's position would be an alternative position if the patient was confined to bed, but sitting in a chair allows better ventilation. The Trendelenburg position or sitting upright in bed with the knees flexed would decrease the patient's ability to ventilate well.

The nurse is reviewing the medical records for five patients who are scheduled for their yearly physical examinations in September. Which patients should receive the inactivated influenza vaccination (select all that apply)? a. A 76-yr-old nursing home resident b. A 36-yr-old female patient who is pregnant c. A 42-yr-old patient who has a 15 pack-year smoking history d. A 30-yr-old patient who takes corticosteroids for rheumatoid arthritis e. A 24-yr-old patient who has allergies to penicillin and cephalosporins

- a 76-year old nursing home resident - a 36-year old female who is pregnant - a 30-yr old patient who takes corticosteroids for rheumatoid arthritis Rationale: Current guidelines suggest that healthy individuals between 6 months and age 49 years receive intranasal immunization with live, attenuated influenza vaccine. Individuals who are pregnant, residents of nursing homes, or are immunocompromised or who have chronic medical conditions should receive inactivated vaccine by injection. The corticosteroid use by the 30-yr-old patient increases the risk for infection.

The clinic nurse is teaching a patient with acute sinusitis. Which interventions should the nurse plan to include in the teaching session (select all that apply)? a. Decongestants can be used to relieve swelling. b. Blowing the nose should be avoided to decrease the nosebleed risk. c. Taking a hot shower will increase sinus drainage and decrease pain. d. Saline nasal spray can be made at home and used to wash out secretions. e. You will be more comfortable if you keep your head in an upright position.

-decongestants can be used to relieve swelling -taking a hot shower will increase sinus drainage and decrease pain -saline nasal spray can be made a t home and used to wash out secretions -you will be more comfortable if you keep your head in an upright position Rationale: The steam and heat from a shower will help thin secretions and improve drainage. Decongestants can be used to relieve swelling. Patients can use either over-the-counter sterile saline solutions or home-prepared saline solutions to thin and remove secretions. Maintaining an upright posture decreases sinus pressure and the resulting pain. Blowing the nose after a hot shower or using the saline spray is recommended to expel secretions.

A young adult patient who denies any history of smoking is seen in the clinic with a new diagnosis of chronic obstructive pulmonary disease (COPD). The nurse should plan to teach the patient about a. 1-antitrypsin testing. b. leukotriene modifiers. c. use of the nicotine patch. d. continuous pulse oximetry.

1-antitrypsin testing Rationale: When COPD occurs in young patients, especially without a smoking history, a genetic deficiency in 1-antitrypsin should be suspected. Because the patient does not smoke, a nicotine patch would not be ordered. There is no indication that the patient requires continuous pulse oximetry. Leukotriene modifiers would be used in patients with asthma, not with COPD

A patient with asthma has a personal best peak expiratory flow rate (PEFR) of 400 L/min. When explaining the asthma action plan, the nurse will teach the patient that a change in therapy is needed when the PEFR is less than ___ L/minute

320 Rationale: A PEFR less than 80% of the personal best indicates that the patient is in the yellow zone where changes in therapy are needed to prevent progression of the airway narrowing.

The nurse monitors a patient in the emergency department after chest tube placement for a hemopneumothorax. The nurse is most concerned if which assessment finding is observed? a. A large air leak in the water-seal chamber b. 400 mL of blood in the collection chamber c. Complaint of pain with each deep inspiration d. Subcutaneous emphysema at the insertion site

400 mL of blood in the collection chamber Rationale: The large amount of blood may indicate that the patient is in danger of developing hypovolemic shock. An air leak would be expected immediately after chest tube placement fora pneumothorax. Initially, brisk bubbling of air occurs in this chamber when a pneumothoraxis evacuated. The pain should be treated but is not as urgent a concern as the possibility of continued hemorrhage. Subcutaneous emphysema should be monitored but is not unusual in a patient with pneumothorax. A small amount of subcutaneous air is harmless and will be reabsorbed.

The nurse receives a change-of-shift report on the following patients with chronic obstructive pulmonary disease (COPD). Which patient should the nurse assess first? a. A patient with loud expiratory wheezes b. A patient with a respiratory rate of 38 breaths/min c. A patient who has a cough productive of thick, green mucus d. A patient with jugular venous distention and peripheral edema

A patient with a respiratory rate of 38 breaths/min Rationale: A respiratory rate of 38/min indicates severe respiratory distress, and the patient needs immediate assessment and intervention to prevent possible respiratory arrest. The other patients also need assessment as soon as possible, but they do not need to be assessed as urgently as the patient with tachypnea.

The nurse reviews the medication administration record (MAR) for a patient having an acute asthma attack. Which medication should the nurse administer first? a. Methylprednisolone (Solu-Medrol) 60 mg IV b. Albuterol (Ventolin HFA) 2.5 mg per nebulizer c. Salmeterol (Serevent) 50 mcg per dry-powder inhaler (DPI) d. Ipratropium (Atrovent) 2 puffs per metered-dose inhaler (MDI)

Albuterol (Ventolin HFA) 2.5 mg per nebulizer Rationale: Albuterol is a rapidly acting bronchodilator and is the first-line medication to reverse airway narrowing in acute asthma attacks. The other medications work more slowly.

A patient with severe chronic obstructive pulmonary disease (COPD) tells the nurse, "I wish I were dead! I'm just a burden on everybody." Based on this information, which nursing diagnosis is most appropriate? a. Complicated grieving related to expectation of death b. Chronic low self-esteem related to physical dependence c. Ineffective coping related to unknown outcome of illness d. Deficient knowledge related to lack of education about COPD

Chronic low self-esteem related to physical dependence Rationale: The patient's statement about not being able to do anything for himself or herself supports this diagnosis. Although deficient knowledge, complicated grieving, and ineffective coping may also be appropriate diagnoses for patients with COPD, the data for this patient do not support these diagnoses.

Which finding in a patient hospitalized with bronchiectasis is most important to report to the health care provider? a. Cough productive of bloody, purulent mucus b. Scattered crackles and wheezes heard bilaterally c. Complaint of sharp chest pain with deep breathing d. Respiratory rate 28 breaths/minute while ambulating

Cough productive of bloody, purulent mucus Rationale: Hemoptysis may indicate life-threatening hemorrhage, and should be reported immediately to the health care provider. The other findings are frequently noted in patients with bronchiectasis and may need further assessment but are not indicators of life-threatening complications

Which assessment finding in a patient who has received omalizumab (Xolair) is most important to report immediately to the health care provider? a. Pain at injection site b. Flushing and dizziness c. Peak flow reading 75% of normal d. Respiratory rate 24 breaths/minute

Flushing and dizziness Rationale: Flushing and dizziness may indicate that the patient is experiencing an an aphylactic reaction, and immediate intervention is needed. The other information should also be reported, but do not indicate possibly life-threatening complications of omalizumab therapy.

Postural drainage with percussion and vibration is ordered twice daily for a patient with chronic bronchitis. Which intervention should the nurse include in the plan of care? a. Schedule the procedure 1 hour after the patient eats. b. Maintain the patient in the lateral position for 20 minutes. c. Give the prescribed albuterol (Ventolin HFA) before the therapy. d. Perform percussion before assisting the patient to the drainage position.

Give the prescribed albuterol (Ventolin HFA) before the therapy Rationale: Bronchodilators are administered before chest physiotherapy. Postural drainage, percussion, and vibration should be done 1 hour before or 3 hours after meals. Patients remain in each postural drainage position for 5 minutes. Percussion is done while the patient is in the postural drainage position.

The nurse provides dietary teaching for a patient with chronic obstructive pulmonary disease (COPD) who has a low body mass index (BMI). Which patient statement indicates that the teaching has been effective? a. "I will drink lots of fluids with my meals." b. "I can have ice cream as a snack every day." c. "I will exercise for 15 minutes before meals." d. "I will decrease my intake of meat and poultry."

I can have ice cream as a snack every day Rationale: High-calorie foods such as ice cream are an appropriate snack for patients with COPD. Fluid intake of 3 L/day is recommended, but fluids should be taken between meals rather than with meals to improve oral intake of solid foods. The patient should avoid exercise for an hour before meals to prevent fatigue while eating. Meat and dairy products are high in protein and are good choices for the patient with COPD.

The nurse discusses management of upper respiratory infections (URI) with a patient who has acute sinusitis. Which statement by the patient indicates that additional teaching is needed? a. "I can take acetaminophen (Tylenol) to treat my discomfort." b. "I will drink lots of juices and other fluids to stay well hydrated." c. "I can use nasal decongestant spray until the congestion is gone." d. "I will watch for changes in nasal secretions or the sputum that I cough up."

I can use nasal decongestant spray until the congestion is gone Rationale: the nurse should clarify that nasal decongestant sprays should be used for no more than 3 days to prevent rebound vasodilation and congestion. the other responses indicate that the teaching has been effective.

The nurse completes discharge instructions for a patient with a total laryngectomy. Which statement by the patient indicates that additional instruction is needed? a. "I must keep the stoma covered with an occlusive dressing." b. "I need to have smoke and carbon monoxide detectors installed." c. "I can participate in my prior fitness activities except swimming." d. "I should wear a Medic-Alert bracelet to identify me as a neck breather."

I must keep the stoma covered with an occlusive dressing Rationale: The stoma may be covered with clothing or a loose dressing, but this is not essential. An occlusive dressing will completely block the patient's airway. The other patient comments are all accurate and indicate that the teaching has been effective.

The nurse completes discharge teaching for a patient who has had a lung transplant. Which patient statement indicates to the nurse that the teaching has been effective? a. "I will make an appointment to see the doctor every year." b. "I will stop taking the prednisone if I experience a dry cough." c. "I will not worry if I feel a little short of breath with exercise." d. "I will call the health care provider right away if I develop a fever."

I will call the health care provider right away if I develop a fever Rationale: Low-grade fever may indicate infection or acute rejection so the patient should notify the health care provider immediately if the temperature is elevated. Patients require frequent follow-up visits with the transplant team. Annual health care provider visits would not be sufficient. Home O2 use is not an expectation after lung transplant. Shortness of breath should be reported. Low-grade fever, fatigue, dyspnea, dry cough, and O2 desaturation are signs of rejection. Immunosuppressive therapy, including prednisone, needs to be continued to prevent rejection.

The nurse provides discharge instructions to a patient who was hospitalized for pneumonia. Which statement, if made by the patient, indicates a good understanding of the instructions? a. "I will call my health care provider if I still feel tired after a week." b. "I will continue to do deep breathing and coughing exercises at home." c. "I will schedule two appointments for the pneumonia and influenza vaccines." d. "I will cancel my follow-up chest x-ray appointment if I feel better next week."

I will continue to do deep breathing and coughing exercises at home Rationale: Patients should continue to cough and deep breathe after discharge. Fatigue is expected for several weeks. The pneumococcal and influenza vaccines can be given at the same time in different arms. A follow-up chest x-ray needs to be done in 6 to 8 weeks to evaluate resolution of pneumonia.

the nurse teaches a patient about discharge instructions after a rhinoplasty. which statement, if made by the patient, indicates that the teaching was successful? a. My nose will look normal after 24-48 hours. b. I can take 800 mg ibuprofen every 6 hours for pain. c. I will remove and reapply the nasal packing every day. d. I will elevate my head for 48 hours to minimize swelling.

I will elevate my head for 48 hours to minimize swelling Rationale: maintaining the head in an elevated position will decrease the amount of nasal swelling. NSAIDs, like ibuprofen, increase the risk for postoperative bleeding and should not be used postoperatively. The patient would not remove or reapply nasal packing, which is usually removed by the surgeon on the day after surgery. Although return to a preinjury appearance is the goal of the surgery, it is not always possible to achieve this result, especially in the first few weeks after surgery.

The nurse teaches a patient about pulmonary spirometry testing. Which statement, if made by the patient, indicates teaching was effective? a. "I should use my inhaler right before the test." b. "I won't eat or drink anything 8 hours before the test." c. "I will inhale deeply and blow out hard during the test." d. "My blood pressure and pulse will be checked every 15 minutes."

I will inhale deeply and blow out hard during the test Rationale: For spirometry, the patient should inhale deeply and exhale as long, hard, and fast as possible. The other actions are not needed. The administration of inhaled bronchodilators should be avoided 6 hours before the procedure.

Which statement by the patient indicates that teaching has been effective for a patient scheduled for radiation therapy of the larynx? a. "I will need to buy a water bottle to carry with me." b. "I should not use any lotions on my neck and throat." c. "Until the radiation is complete, I may have diarrhea." d. "Alcohol-based mouthwashes will help clean my mouth."

I will need to buy a water bottle to carry with me Rationale: Xerostomia can be partially alleviated by drinking fluids at frequent intervals. Radiation will damage tissues at the site being radiated but should not affect the abdominal organs, so loose stools are not a usual complication of head and neck radiation therapy. Frequent oral rinsing with non-alcohol-based rinses is recommended. Prescribed lotions and sunscreen may be used on radiated skin, although they should not be used just before the radiation therapy.

The nurse provides discharge teaching for a patient who has two fractured ribs from an automobile accident. Which statement, if made by the patient, would indicate that teaching has been effective? a. "I am going to buy a rib binder to wear during the day." b. "I can take shallow breaths to prevent my chest from hurting." c. "I should plan on taking the pain pills only at bedtime so I can sleep." d. "I will use the incentive spirometer every hour or two during the day."

I will use the incentive spirometer every hour or two during the day Rationale: Prevention of the complications of atelectasis & pneumonia is a priority after rib fracture. This can be ensured by deep breathing & coughing. Use of a rib binder, shallow breathing, & taking pain medications only at night are likely to result in atelectasis

The nurse admits a patient who has a diagnosis of an acute asthma attack. Which statement indicates that the patient may need teaching regarding medication use? a. "I have not had any acute asthma attacks during the past year." b. "I became short of breath an hour before coming to the hospital." c. "I've been taking Tylenol 650 mg every 6 hours for chest wall pain." d. "I've been using my albuterol inhaler more frequently over the last 4 days."

I've been using my albuterol inhaler more frequently over the last 4 days Rationale: The increased need for a rapid-acting bronchodilator should alert the patient that an acute attack may be imminent and that a change in therapy may be needed. The patient should be taught to contact a health care provider if this occurs. The other data do not indicate any need for additional teaching.

The clinic nurse makes a follow-up telephone call to a patient with asthma. The patient reports having a baseline peak flow reading of 600 L/min, and the current peak flow is 420 L/min. Which action should the nurse take first? a. Tell the patient to go to the hospital emergency department. b. Instruct the patient to use the prescribed albuterol (Ventolin HFA). c. Ask about recent exposure to any new allergens or asthma triggers. d. Question the patient about use of the prescribed inhaled corticosteroids.

Instruct the patient to use the prescribed albuterol (Ventolin HFA) Rationale: The patient's peak flow is 70% of normal, indicating a need for immediate use of short-acting 2-adrenergic SABA medications. Assessing for correct use of medications or exposure to allergens is also appropriate, but would not address the current decrease in peak flow. Because the patient is currently in the yellow zone, hospitalization is not needed.

A patient is receiving 35% O2 via a Venturi mask. To ensure the correct amount of O2 delivery, which action by the nurse is important? a. Teach the patient to keep the mask on during meals. b. Keep the air entrainment ports clean and unobstructed. c. Give a high enough flow rate to keep the bag from collapsing. d. Drain moisture condensation from the corrugated tubing every hour.

Keep the air entrainment ports clean and unobstructed Rationale: The air entrainment ports regulate the O2 percentage delivered to the patient, so they must be unobstructed. The other options refer to other types of O2 devices. A high O2 flow rate is needed when giving O2 by partial rebreather or nonrebreather masks. Draining O2 tubing is necessary when caring for a patient receiving mechanical ventilation. The mask can be removed or changed to a nasal cannula at a prescribed setting when the patient eats.

A diabetic patient's arterial blood gas (ABG) results are pH 7.28; PaCO2 34 mm Hg; PaO2 85 mm Hg; HCO3- 18 mEq/L. The nurse would expect which finding? a. Intercostal retractions b. Kussmaul respirations c. Low oxygen saturation (SpO2) d. Decreased venous O2 pressure

Kussmaul respirations Rationale: Kussmaul (deep and rapid) respirations are a compensatory mechanism for metabolic acidosis. The low pH and low bicarbonate result indicate metabolic acidosis. Intercostal retractions, a low oxygen saturation rate, and a decrease in venous O2 pressure would not be caused by acidosis.

A patient who is experiencing an acute asthma attack is admitted to the emergency department. Which assessment should the nurse complete first? a. Listen to the patient's breath sounds. b. Ask about inhaled corticosteroid use. c. Determine when the dyspnea started. d. Obtain the forced expiratory volume (FEV) flow rate.

Listen to the patient's breath sounds Rationale: Assessment of the patient's breath sounds will help determine how effectively the patient is ventilating and whether rapid intubation may be necessary. The length of time the attack has persisted is not as important as determining the patient's status at present. Most patients having an acute attack will be unable to cooperate with an FEV measurement. It is important to know about the medications the patient is using but not as important as assessing the breath sounds.

A patient who has been experiencing an asthma attack develops bradycardia and a decrease in wheezing. Which action should the nurse take first? a. Notify the health care provider. b. Document changes in respiratory status. c. Encourage the patient to cough and deep breathe. d. Administer IV methylprednisolone (Solu-Medrol).

Notify the health care provider Rationale: The patient's assessment indicates impending respiratory failure, and the nurse should prepare to assist with intubation and mechanical ventilation after notifying the health care provider. IV corticosteroids require several hours before having any effect on respiratory status. The patient will not be able to cough or deep breathe effectively. Documentation is not a priority at this time

A patient hospitalized with chronic obstructive pulmonary disease (COPD) is being discharged home on O2 therapy. Which instruction should the nurse include in the discharge teaching? a. Travel is not possible with the use of O2 devices. b. O2 flow should be increased if the patient has more dyspnea. c. O2 use can improve the patient's prognosis and quality of life. d. Storage of O2 requires large metals tanks that each last 4 to 6 hours.

O2 use can improve the patient's prognosis and quality of life Rationale: The use of home O2 improves quality of life and prognosis. Because increased dyspnea maybe a symptom of an acute process such as pneumonia, the patient should notify the health care provider rather than increasing the O2 flow rate if dyspnea becomes worse. O2 can be supplied using liquid, storage tanks, or concentrators, depending on individual patient circumstances.Travel is possible using portable O2 concentrators.

Which nursing action for a patient with chronic obstructive pulmonary disease (COPD) could the nurse delegate to experienced unlicensed assistive personnel (UAP)? a. Obtain O2 saturation using pulse oximetry. b. Monitor for increased O2 need with exercise. c. Teach the patient about safe use of O2 at home. d. Adjust O2 to keep saturation in prescribed parameters.

Obtain O2 saturation using pulse oximetry Rationale: UAP can obtain O2 saturation (after being trained and evaluated in the skill). The other actions require more education and a scope of practice that licensed practical/vocational nurses (LPN/LVNs) or registered nurses (RNs) would have.

The nurse is caring for a patient who has just had a thoracentesis. Which assessment information obtained by the nurse is a priority to communicate to the health care provider? a. O2 saturation is 88%. b. Blood pressure is 155/90 mm Hg. c. Pain level is 5 (on 0 to 10 scale) with a deep breath. d. Respiratory rate is 24 breaths/minute when lying flat.

O2 saturation is 88% Rationale: O2 saturation would be expected to improve after a thoracentesis. A saturation of 88% indicates that a complication such as pneumothorax may be occurring. The other assessment data also indicate a need for ongoing assessment or intervention, but the low O2 saturation is the priority

The emergency department nurse is evaluating the effectiveness of therapy for a patient who has received treatment during an asthma attack. Which assessment finding is the best indicator that the therapy has been effective? a. No wheezes are audible. b. O2 saturation is >90%. c. Accessory muscle use has decreased. d. Respiratory rate is 16 breaths/minute

O2 saturation is >90% Rationale: The goal for treatment of an asthma attack is to keep the O2 saturation above 90%. The other patient data may occur when the patient is too fatigued to continue with the increased work of breathing required in an asthma attack.

A patient newly diagnosed with asthma is being discharged. The nurse anticipates including which topic in the discharge teaching? a. Use of long-acting -adrenergic medications b. Side effects of sustained-release theophylline c. Self-administration of inhaled corticosteroids d. Complications associated with O2 therapy

Self-administration of inhaled corticosteroids Rationale: Inhaled corticosteroids are more effective in improving asthma than any other drug and are indicated for all patients with persistent asthma. The other therapies would not typically be first-line treatments for newly diagnosed asthma

A patient with cystic fibrosis (CF) has blood glucose levels that are consistently between 180 to 250 mg/dL. Which nursing action will the nurse plan to implement? a. Discuss the role of diet in blood glucose control. b. Evaluate the patient's use of pancreatic enzymes. c. Teach the patient about administration of insulin. d. Give oral hypoglycemic medications before meals.

Teach the patient about administration of insulin Rationale: The glucose levels indicate that the patient has developed CF-related diabetes, and insulin therapy is required. Because the etiology of diabetes in CF is inadequate insulin production,oral hypoglycemic agents are not effective. Patients with CF need a high-calorie diet.Inappropriate use of pancreatic enzymes would not be a cause of hyperglycemia in a patient with CF.

A patient with chronic obstructive pulmonary disease (COPD) has coarse crackles throughout the lung fields and a chronic, nonproductive cough. Which nursing intervention will be most effective? a. Change the O2 flow rate to the highest prescribed rate. b. Teach the patient to use the Flutter airway clearance device. c. Reinforce the ongoing use of pursed-lip breathing techniques. d. Teach the patient about consistent use of inhaled corticosteroids.

Teach the patient to use the Flutter airway clearance device Rationale: Airway clearance devices assist with moving mucus into larger airways, where it can more easily be expectorated. The other actions may be appropriate for some patients with COPD, but they are not indicated for this patient's problem of thick mucus secretions.

The nurse takes an admission history on a patient with possible asthma who has new-onset wheezing and shortness of breath. Which information may indicate a need for a change in therapy? a. The patient has chronic inflammatory bowel disease. b. The patient has a history of pneumonia 6 months ago. c. The patient takes propranolol (Inderal) for hypertension. d. The patient uses acetaminophen (Tylenol) for headaches.

The patient takes propranolol (Inderal) for hypertension Rationale: b-Blockers such as propranolol can cause bronchospasm in some patients with asthma. The other information will be documented in the health history but does not indicate a need for a change in therapy

The nurse teaches a patient who has asthma about peak flow meter use. Which action by the patient indicates that teaching was successful? a. The patient inhales rapidly through the peak flow meter mouthpiece. b. The patient takes montelukast (Singulair) for peak flows in the red zone. c. The patient calls the health care provider when the peak flow is in the green zone. d. The patient uses an albuterol (Ventolin HFA) inhaler for peak flows in the yellow zone

The patient uses an albuterol (ventolin HFA) inhaler for peak flows in the yellow zone Rationale: Readings in the yellow zone indicate a decrease in peak flow. The patient should use short-acting 2-adrenergic (SABA) medications. Readings in the green zone indicate good asthma control. The patient should exhale quickly and forcefully through the peak flow meter mouthpiece to obtain the readings. Readings in the red zone do not indicate good peak flow, and the patient should take a fast-acting bronchodilator and call the health care provider for further instructions. Singulair is not indicated for acute attacks but rather is used for maintenance therapy.

The nurse completes an admission assessment on a patient with asthma. Which information given by patient is indicates a need for a change in therapy? a. The patient uses albuterol (Ventolin HFA) before aerobic exercise. b. The patient says that the asthma symptoms are worse every spring. c. The patient's heart rate increases after using the albuterol (Ventolin HFA) inhaler. d. The patient's only medications are albuterol (Ventolin HFAl) and salmeterol (Serevent).

The patient's only medications are albuterol (Ventolin HFAl) and salmeterol (Serevent) Rationale: Long-acting 2-agonists should be used only in patients who also are using an inhaled corticosteroid for long-term control. Salmeterol should not be used as the first-line therapy for long-term control. Using a bronchodilator before exercise is appropriate. The other information given by the patient requires further assessment by the nurse but is not unusual for a patient with asthma.

The nurse supervises unlicensed assistive personnel (UAP) who are providing care for a patient with right lower lobe pneumonia. The nurse should intervene if which action by UAP is observed? a. UAP assist the patient to ambulate to the bathroom. b. UAP help splint the patient's chest during coughing. c. UAP transfer the patient to a bedside chair for meals. d. UAP lower the head of the patient's bed to 15 degrees.

UAP lower the head of the patient's bed to 15 degrees Rationale: Positioning the patient with the head of the bed lowered will decrease ventilation. The other actions are appropriate for a patient with pneumonia.

The nurse assesses a patient with a history of asthma. Which assessment finding indicates that the nurse should take immediate action? a. Pulse oximetry reading of 91% b. Respiratory rate of 26 breaths/min c. Use of accessory muscles in breathing d. Peak expiratory flow rate of 240 L/min

Use of accessory muscles in breathing Rationale: Use of accessory muscle indicates that the patient is experiencing respiratory distress, and rapid intervention is needed. The other data indicate the need for ongoing monitoring and assessment but do not suggest that immediate treatment is required.

Which instruction should the nurse include in an exercise teaching plan for a patient with chronic obstructive pulmonary disease (COPD)? a. "Avoid upper body exercise to prevent dyspnea." b. "Stop exercising if you start to feel short of breath." c. "Use the bronchodilator before you start to exercise." d. "Breathe in and out through the mouth while you exercise."

Use the bronchodilator before you start to exercise Rationale: Use of a bronchodilator before exercise improves airflow for some patients and is recommended. Shortness of breath is normal with exercise and not a reason to stop. Patients should be taught to breathe in through the nose and out through the mouth (using a pursed-lip technique). Upper-body exercise can improve the mechanics of breathing in patients with COPD.

The nurse develops a teaching plan to help increase activity tolerance at home for an older adult with severe chronic obstructive pulmonary disease (COPD). Which instructions would be appropriate for the nurse to include in the plan of care? a. Stop exercising when you feel short of breath. b. Walk until pulse rate exceeds 130 beats/minute. c. Limit exercise to activities of daily living (ADLs). d. Walk 15 to 20 minutes a day at least 3 times/week.

Walk 15 to 20 minutes a day at least 3 times/week Rationale: Encourage the patient to walk 15 to 20 minutes a day at least three times a week with gradual increases. Shortness of breath is normal with exercise and not an indication that the patient should stop. Limiting exercise to ADLs will not improve the patient's exercise tolerance. A 70-yr-old patient should have a pulse rate of 120 beats/min or less with exercise (80% of the maximal heart rate of 150 beats/min ).

A young adult female patient with cystic fibrosis (CF) tells the nurse that she is considering getting married and wondering about having children. Which initial response by the nurse is best? a. "Are you aware of the normal lifespan for patients with CF?" b. "Would like more information to help you with that decision?" c. "Many women with CF do not have difficulty conceiving children." d. "You will need to have genetic counseling before making a decision."

Would like more information to help you with that decision Rationale: The nurse's initial response should be to assess the patient's knowledge level and need for information. Although the lifespan for patients with CF is likely to be shorter than normal, it would not be appropriate for the nurse to address this as the initial response to the patient's comments. The other responses have accurate information, but the nurse should first assess the patient's understanding about the issues surrounding pregnancy.

The nurse receives change-of-shift report on the following four patients. Which patient should the nurse assess first? a. A 23-yr-old patient with cystic fibrosis who has pulmonary function testing scheduled b. A 46-yr-old patient on bed rest who is complaining of sudden onset of shortness of breath c. A 77-yr-old patient with tuberculosis (TB) who has four medications due in 15 minutes d. A 35-yr-old patient who was admitted with pneumonia and has a temperature of 100.2° F (37.8° C)

a 46-yr-old patient on bed rest who is complaining of sudden onset shortness of breath Rationale: Patients on bed rest who are immobile are at high risk for deep vein thrombosis (DVT). Sudden onset of shortness of breath in a patient with a DVT suggests a pulmonary embolism and requires immediate assessment and action such as O2 administration. The other patients should also be assessed as soon as possible, but there is no indication that they may need immediate action to prevent clinical deterioration.

After change-of-shift report, which patient should the nurse assess first? a. A 72-yr-old with cor pulmonale who has 4+ bilateral edema in his legs and feet b. A 28-yr-old with a history of a lung transplant and a temperature of 101° F (38.3° C) c. A 40-yr-old with a pleural effusion who is complaining of severe stabbing chest pain d. A 64-yr-old with lung cancer and tracheal deviation after subclavian catheter insertion

a 64-yr-old with lung cancer and tracheal deviation after subclavian catheter insertion The patient's history and symptoms suggest possible tension pneumothorax, a medical emergency. The other patients also require assessment as soon as possible, but tension pneumothorax will require immediate treatment to avoid death from inadequate cardiac output or hypoxemia.

Which patient in the ear, nose, and throat (ENT) clinic should the nurse assess first? a. A patient who is complaining of a sore throat and has a muffled voice b. A patient who has a "scratchy throat" and a positive rapid strep antigen test c. A patient who is receiving radiation for throat cancer and has severe fatigue d. A patient with a history of a total laryngectomy whose stoma is red and inflamed

a patient who is complaining of a sore throat and has a muffled voice Rationale: The patient's clinical manifestation of a muffled voice suggests a possible peritonsillar abscess that could lead to an airway obstruction requiring rapid assessment and potential treatment. The other patients do not have diagnoses or symptoms that indicate any life-threatening problems.

After the nurse has received change-of-shift report, which patient should the nurse assess first? a. A patient with pneumonia who has crackles in the right lung base b. A patient with chronic bronchitis who has a low forced vital capacity c. A patient with possible lung cancer who has just returned after bronchoscopy d. A patient with hemoptysis and a 16-mm induration after tuberculin skin testing

a patient with possible lung cancer who has just returned after bronchoscopy Rationale: Because the cough and gag are decreased after bronchoscopy, this patient should be assessed for airway patency. The other patients do not have clinical manifestations or procedures that require immediate assessment by the nurse.

The nurse supervises a student nurse who is assigned to take care of a patient with active tuberculosis (TB). Which action, if performed by the student nurse, would require an intervention by the nurse? a. The patient is offered a tissue from the box at the bedside. b. A surgical face mask is applied before visiting the patient. c. A snack is brought to the patient from the unit refrigerator. d. Hand washing is performed before entering the patient's room.

a surgical face mask is applied before visiting the patient Rationale: A high-efficiency particulate-absorbing (HEPA) mask, rather than a standard surgical mask, should be used when entering the patient's room because the HEPA mask can filter out 100% of small airborne particles. Hand washing before entering the patient's room is appropriate. Because anorexia and weight loss are frequent problems in patients with TB, bringing food to the patient is appropriate. The student nurse should perform hand washing after handling a tissue that the patient has used, but no precautions are necessary when giving the patient an unused tissue

The nurse observes that a patient with respiratory disease experiences a decrease in SpO2 from 93% to 85% while the patient is ambulating. What is the priority action of the nurse? a. Notify the health care provider. b. Administer PRN supplemental O2. c. Document the response to exercise. d. Encourage the patient to pace activity.

administer PRN supplemental O2 Rationale: The drop in SpO2 to 85% indicates that the patient is hypoxemic and needs supplemental O2 when exercising. The other actions are also important, but the first action should be to correct the hypoxemia

A patient seen in the asthma clinic has recorded daily peak flow rates that are 75% of the baseline. Which action will the nurse plan to take next? a. Increase the dose of the leukotriene inhibitor. b. Teach the patient about the use of oral corticosteroids. c. Administer a bronchodilator and recheck the peak flow. d. Instruct the patient to keep the scheduled follow-up appointment.

administer a bronchodilator and recheck the peak flow Rationale: The patient's peak flow reading indicates that the condition is worsening (yellow zone). The patient should take the bronchodilator and recheck the peak flow. Depending on whether the patient returns to the green zone, indicating well-controlled symptoms, the patient may be prescribed oral corticosteroids or a change in dosing of other medications. Keeping the next appointment is appropriate, but the patient also needs to be taught how to control symptoms now and use the bronchodilator

An hour after a thoracotomy, a patient complains of incisional pain at a level 7 (based on 0 to 10 scale) and has decreased left-sided breath sounds. The pleural drainage system has 100 mL of bloody drainage and a large air leak. Which action should the nurse take? a. Clamp the chest tube in two places. b. Administer the prescribed morphine. c. Milk the chest tube to remove any clots. d. Assist the patient with incentive spirometry.

administer the prescribed morphine Rationale: Treat the pain. The patient is unlikely to take deep breaths or cough until the pain level is lower. A chest tube output of 100 mL is not unusual in the first hour after thoracotomy. Milking or stripping chest tubes is no longer recommended because these practices can dangerously increase intrapleural pressures and damage lung tissues. Position tubing so that drainage flows freely to negate need for milking or stripping. An air leak is expected in the initial postoperative period after thoracotomy. Clamping the chest tube is not indicated and may lead to dangerous development of a tension pneumothorax.

Which factors will the nurse consider when calculating the CURB-65 score for a patient with pneumonia (select all that apply)? a. Age b. Blood pressure c. Respiratory rate d. O2 saturation e. Presence of confusion f. Blood urea nitrogen (BUN) level

age blood pressure respiratory rate presence of confusion blood urea nitrogen (BUN) level Rationale: Data collected for the CURB-65 are mental status (confusion), BUN (elevated), blood pressure (decreased), respiratory rate (increased), and age (65 years and older). The other information is also essential to assess, but are not used for CURB-65 scoring.

The nurse is caring for a patient who has a right-sided chest tube after a right lower lobectomy. Which nursing action can the nurse delegate to the unlicensed assistive personnel (UAP)? a. Document the amount of drainage every 8 hours. b. Obtain samples of drainage for culture from the system. c. Assess patient pain level associated with the chest tube. d. Check the water-seal chamber for the correct fluid level.

document the amount of drainage every 8 hours Rationale: UAP education includes documentation of intake and output. The other actions are within the scope of practice and education of licensed nursing personnel.

A patient with acute dyspnea is scheduled for a spiral computed tomography (CT) scan. Which information obtained by the nurse is a priority to communicate to the health care provider before the CT? a. Allergy to shellfish b. Respiratory rate of 30 c. Apical pulse of 104 d. O2 saturation of 90%

allergy to shellfish Rationale: Because iodine-based contrast media is used during a spiral CT, the patient may need to have the CT scan without contrast or be premedicated before injection of the contrast media. The increased pulse, low oxygen saturation, and tachypnea all indicate a need for further assessment or intervention but do not indicate a need to modify the CT procedure.

A patient is scheduled for a computed tomography (CT) scan of the chest with contrast media. Which assessment findings should the nurse report to the health care provider before the patient goes for the CT? (select all that apply) a. Allergy to shellfish b. Patient reports claustrophobia c. Elevated serum creatinine level d. Recent bronchodilator inhaler use e. Inability to remove a wedding band

allergy to shellfish elevated serum creatinine level Rationale: Because the contrast media is iodine based and may cause dehydration and decreased renal blood flow, asking about iodine allergies (such as allergy to shellfish) and monitoring renal function before the CT scan are necessary. The other actions are not contraindications for CT of the chest, although they may be for other diagnostic tests, such as magnetic resonance imaging or pulmonary spirometry

Which action should the nurse take first when a patient develops epistaxis? a. Pack the affected nare tightly with an epistaxis balloon. b. Apply squeezing pressure to the nostrils for 10 minutes. c. Obtain silver nitrate that may be needed for cauterization. d. Instill a vasoconstrictor medication into the affected nare.

apply squeezing pressure to the nostrils for 10 minutes Rationale: The first nursing action for epistaxis is to apply direct pressure by pinching the nostrils. Application of cold packs may decrease blood flow to the area but will not be sufficient to stop bleeding. Cauterization, nasal packing, and vasoconstrictors are medical interventions that may be needed if pressure to the nares does not stop the bleeding, but these are not the first actions to take for a nosebleed.

A patient has acute bronchitis with a nonproductive cough and wheezes. Which topic should the nurse plan to include in the teaching plan? a. Purpose of antibiotic therapy b. Ways to limit oral fluid intake c. Appropriate use of cough suppressants d. Safety concerns with home O2 therapy

appropriate use of cough suppressants Rationale: Cough suppressants are frequently prescribed for acute bronchitis. Because most acute bronchitis is viral in origin, antibiotics are not prescribed unless there are systemic symptoms. Fluid intake is encouraged. Home O2 is not prescribed for acute bronchitis, although it may be used for chronic bronchitis.

A patient diagnosed with active tuberculosis (TB) is homeless and has a history of alcohol abuse. Which intervention by the nurse will be most effective in ensuring adherence with the treatment regimen? a. Repeat warnings about the high risk for infecting others several times. b. Give the patient written instructions about how to take the medications. c. Arrange for a daily meal and drug administration at a community center. d. Arrange for the patient's friend to administer the medication on schedule.

arrange for a daily meal and drug administration at a community center Rationale: Directly observed therapy is the most effective means for ensuring compliance with the treatment regimen, and arranging a daily meal will help ensure that the patient is available to receive the medication. The other nursing interventions may be appropriate for some patients but are not likely to be as helpful for this patient's situation

A patient who has a history of chronic obstructive pulmonary disease (COPD) was hospitalized for increasing shortness of breath and chronic hypoxemia (SaO2 levels of 89% to 90%). In planning for discharge, which action by the nurse will be most effective in improving compliance with discharge teaching? a. Have the patient repeat the instructions immediately after teaching. b. Accomplish the patient teaching just before the scheduled discharge. c. Arrange for the patient's caregiver to be present during the teaching. d. Start giving the patient discharge teaching during the admission process.

arrange for the patient's caregiver to be present during the teaching Rationale: Hypoxemia interferes with the patient's ability to learn and retain information, so having the patient's caregiver present will increase the likelihood that discharge instructions will be followed. Having the patient repeat the instructions will indicate that the information is understood at the time, but it does not guarantee retention of the information. Because the patient is likely to be distracted just before discharge, giving discharge instructions just before discharge is not ideal. The patient is likely to be anxious and even more hypoxemic than usual on the day of admission, so teaching about discharge should be postponed.

After 2 months of tuberculosis (TB) treatment with isoniazid, rifampin (Rifadin), pyrazinamide, and ethambutol, a patient continues to have positive sputum smears for acid-fast bacilli (AFB). Which action should the nurse take next? a. Teach about drug-resistant TB. b. Schedule directly observed therapy. c. Ask the patient whether medications have been taken as directed. d. Discuss the need for an injectable antibiotic with the health care provider.

ask the patient whether medications have been taken as directed Rationale: The first action should be to determine whether the patient has been compliant with drug therapy because negative sputum smears would be expected if the TB bacillus is susceptible to the medications and if the medications have been taken correctly. Assessment is the first step in the nursing process. Depending on whether the patient has been compliant or not,different medications or directly observed therapy may be indicated. The other options are interventions based on assumptions until an assessment has been completed.

A patient with a tracheostomy has a new order for a fenestrated tracheostomy tube. Which action should the nurse include in the plan of care in collaboration with the speech therapist? a. Leave the tracheostomy inner cannula inserted at all times. b. Place the decannulation cap in the tube before cuff deflation. c. Assess the ability to swallow before using the fenestrated tube. d. Inflate the tracheostomy cuff during use of the fenestrated tube.

assess the ability to swallow before using the fenestrated tube Rationale: Because the cuff is deflated when using a fenestrated tube, the patient's risk for aspiration should be assessed before changing to a fenestrated tracheostomy tube. The decannulation cap is never inserted before cuff deflation because to do so would obstruct the patient's airway.The cuff is deflated and the inner cannula removed to allow air to flow across the patient's vocal cords when using a fenestrated tube.

A patient with bacterial pneumonia has coarse crackles and thick sputum. Which action should the nurse plan to promote airway clearance? a. Restrict oral fluids during the day. b. Teach pursed-lip breathing technique. c. Assist the patient to splint the chest when coughing. d. Encourage the patient to wear the nasal O2 cannula.

assist the patient to splint the chest when coughing Rationale: Coughing is less painful and more likely to be effective when the patient splints the chest during coughing. Fluids should be encouraged to help liquefy secretions. Nasal O2 will improve gas exchange, but will not improve airway clearance. Pursed-lip breathing is used to improve gas exchange in patients with chronic obstructive pulmonary disease but will not improve airway clearance

After a laryngectomy, a patient coughs violently during suctioning and dislodges the tracheostomy tube. Which action should the nurse take first? a. Arrange for arterial blood gases to be drawn immediately. b. Cover stoma with sterile gauze and ventilate through stoma. c. Attempt to reinsert the tracheostomy tube with the obturator in place. d. Assess the patient's oxygen saturation and notify the health care provider

attempt to reinsert the tracheostomy tube with the obturator in place Rationale: The first action should be to attempt to reinsert the tracheostomy tube to maintain the patient's airway. Covering the stoma with a dressing and manually ventilating the patient may be an appropriate action if the nurse is unable to reinsert the tracheostomy tube. Assessing the patient's oxygenation is an important action, but it is not as appropriate until there is an established airway.

The nurse palpates the posterior chest while the patient says "99" and notes absent fremitus. Which action should the nurse take next? a. Palpate the anterior chest and observe for barrel chest. b. Encourage the patient to turn, cough, and deep breathe. c. Review the chest x-ray report for evidence of pneumonia. d. Auscultate anterior and posterior breath sounds bilaterally.

auscultate anterior and posterior breath sounds bilaterally Rationale: To assess for tactile fremitus, the nurse should use the palms of the hands to assess for vibration when the patient repeats a word or phrase such as "99." After noting absent fremitus, the nurse should then auscultate the lungs to assess for the presence or absence of breath sounds. Absent fremitus may be noted with pneumothorax or atelectasis. The vibration is increased in conditions such as pneumonia, lung tumors, thick bronchial secretions, and pleural effusion. Turning, coughing, and deep breathing is an appropriate intervention for atelectasis, but the nurse needs to first assess breath sounds. Fremitus is decreased if the hand is farther from the lung or the lung is hyperinflated (barrel chest). The anterior of the chest is more difficult to palpate for fremitus because of the presence of large muscles and breast tissue.

A patient who was admitted the previous day with pneumonia complains of a sharp pain of 7 (on 0 to 10 scale) "whenever I take a deep breath." Which action will the nurse take next? a. Auscultate for breath sounds. b. Administer the PRN morphine. c. Have the patient cough forcefully. d. Notify the patient's health care provider.

auscultate for breath sounds Rationale: The patient's statement indicates that pleurisy or a pleural effusion may have developed and the nurse will need to listen for a pleural friction rub and decreased breath sounds. Assessment should occur before administration of pain medications. The patient is unlikely to be able to cough forcefully until pain medication has been administered. The nurse will want to obtain more assessment data before calling the health care provider.

Which assessment finding indicates that the nurse should take immediate action for an older patient? a. Weak cough effort b. Barrel-shaped chest c. Dry mucous membranes d. Bilateral basilar crackles

bilateral basilar crackles Rationale: Crackles in the lower half of the lungs indicate that the patient may have an acute problem such as heart failure. The nurse should immediately accomplish further assessments, such as O2 saturation, and notify the health care provider. A barrel-shaped chest, hyperresonance to percussion, and a weak cough effort are associated with aging. Further evaluation may be needed, but immediate action is not indicated. An older patient has a less forceful cough and fewer and less functional cilia. Mucous membranes tend to be drier.

A patient has just been admitted with probable bacterial pneumonia and sepsis. Which order should the nurse implement first? a. Chest x-ray via stretcher b. Blood cultures from two sites c. Ciprofloxacin (Cipro) 400 mg IV d. Acetaminophen (Tylenol) rectal suppository

blood cultures from two sites Rationale: Initiating antibiotic therapy rapidly is essential, but it is important that the cultures be obtained before antibiotic administration. The chest x-ray and acetaminophen administration can be done last.

A patient with acute shortness of breath is admitted to the hospital. Which action should the nurse take during the initial assessment of the patient? a. Ask the patient to lie down to complete a full physical assessment. b. Briefly ask specific questions about this episode of respiratory distress. c. Complete the admission database to check for allergies before treatment. d. Delay the physical assessment to first complete pulmonary function tests

briefly ask specific questions about this episode of respiratory distress Rationale: When a patient has severe respiratory distress, only information pertinent to the current episode is obtained, and a more thorough assessment is deferred until later. Obtaining a comprehensive health history or full physical examination is unnecessary until the acute distress has resolved. Brief questioning and a focused physical assessment should be done rapidly to help determine the cause of the distress and suggest treatment. Checking for allergies is important, but it is not appropriate to complete the entire admission database at this time. The initial respiratory assessment must be completed before any diagnostic tests or interventions can be ordered.

A patient with newly diagnosed lung cancer tells the nurse, "I don't think I'm going to live to see my next birthday." Which is the best initial response by the nurse? a. "Are you ready to talk with your family members about dying now?" b. "Would you like to talk to the hospital chaplain about your feelings?" c. "Can you tell me what it is that makes you think you will die so soon?" d. "Do you think that taking an antidepressant medication would be helpful?"

can you tell me what it is that makes you think you will die so soon? Rationale: The nurse's initial response should be to collect more assessment data about the patient's statement. The answer beginning "Can you tell me what it is" is the most open-ended question and will offer the best opportunity for obtaining more data. The remaining answers offer interventions that may be helpful to the patient, but more assessment is needed to determine whether these interventions are appropriate.

After being hit by a baseball, a patient arrives in the emergency department with a possible nasal fracture. Which finding by the nurse is most important to report to the health care provider? a. Clear nasal drainage b. Complaint of nasal pain c. Bilateral nose swelling and bruising d. Inability to breathe through the nose

clear nasal drainage Rationale: Clear nasal drainage may indicate a meningeal tear with leakage of cerebrospinal fluid. This would place the patient at risk for complications such as meningitis. The other findings are typical with a nasal fracture and do not indicate any complications.

The nurse is performing tuberculosis (TB) skin tests in a clinic that has many patients who have immigrated to the United States. Which question is most important for the nurse to ask before the skin test? a. "Do you take any over-the-counter (OTC) medications?" b. "Do you have any family members with a history of TB?" c. "How long has it been since you moved to the United States?" d. "Did you receive the bacille Calmette-Guérin (BCG) vaccine for TB?"

did you receive the bacille Calmette-Geurin (BCG) vaccine for TB? Rationale: Patients who have received the BCG vaccine will have a positive Mantoux test. Another method for screening (e.g., chest x-ray) will need to be used in determining whether the patient has a TB infection. The other information also may be valuable but is not as pertinent to the decision about doing TB skin testing.

The nurse obtains the following assessment data on an older patient who has influenza. Which information will be most important for the nurse to communicate to the health care provider? a. Fever of 100.4° F (38° C) b. Diffuse crackles in the lungs c. Sore throat and frequent cough d. Myalgia and persistent headache

diffuse crackles in the lungs Rationale: The crackles indicate that the patient may be developing pneumonia, a common complication of influenza, which would require aggressive treatment. Myalgia, headache, mild temperature elevation, and sore throat with cough are typical manifestations of influenza and are treated with supportive care measures such as over-the-counter pain relievers and increased fluid intake.

The nurse completes a shift assessment on a patient admitted in the early phase of heart failure. When auscultating the patient's lungs, which finding would the nurse most likely hear? a. Continuous rumbling, snoring, or rattling sounds mainly on expiration b. Continuous high-pitched musical sounds on inspiration and expiration c. Discontinuous, high-pitched sounds of short duration during inspiration d. A series of long-duration, discontinuous, low-pitched sounds during inspiration

discontinuous, high-pitched musical sounds on inspiration and expiration Rationale: Fine crackles are likely to be heard in the early phase of heart failure. Fine crackles are discontinuous, high-pitched sounds of short duration heard on inspiration. Course crackles are a series of long-duration, discontinuous, low-pitched sounds during inspiration. Wheezes are continuous high-pitched musical sounds on inspiration and expiration.

A patient with a possible pulmonary embolism complains of chest pain and difficulty breathing. The nurse finds a heart rate of 142 beats/min, blood pressure of 100/60 mm Hg, and respirations of 42 breaths/min. Which action should the nurse take first? a. Administer anticoagulant drug therapy. b. Notify the patient's health care provider. c. Prepare patient for a spiral computed tomography (CT). d. Elevate the head of the bed to a semi-Fowler's position

elevate the head of the bed to a semi-Fowler's position Rationale: The patient has symptoms consistent with a pulmonary embolism (PE). Elevating the head of the bed will improve ventilation and gas exchange. The other actions can be accomplished after the head is elevated (and O2 is started). A spiral CT may be ordered by the health care provider to identify PE. Anticoagulants may be ordered after confirmation of the diagnosis of PE.

A patient who is taking rifampin (Rifadin) for tuberculosis calls the clinic and reports having orange discolored urine and tears. Which response by the nurse reflects accurate knowledge about the medication and the patient's illness? a. Ask the patient about any visual changes in red-green color discrimination. b. Question the patient about experiencing shortness of breath, hives, or itching. c. Explain that orange discolored urine and tears are normal while taking this medication. d. Advise the patient to stop the drug and report the symptoms to the health care provider.

explain that orange discolored urine and tears are normal while taking this medication Rationale: Orange-colored body secretions are a side effect of rifampin. The patient does not have to stop taking the medication. The findings are not indicative of an allergic reaction. Alterations in red-green color discrimination commonly occurs when taking ethambutol, which is a different tuberculosis medication.

A patient in the clinic with cystic fibrosis (CF) reports increased sweating and weakness during the summer months. Which action by the nurse would be most appropriate? a. Teach the patient signs of hypoglycemia. b. Have the patient add dietary salt to meals. c. Suggest decreasing intake of dietary fat and calories. d. Instruct the patient about pancreatic enzyme replacements.

have the patient add dietary salt to meals Rationale: Added dietary salt is indicated whenever sweating is excessive, such as during hot weather,when fever is present, or from intense physical activity. The management of pancreatic insufficiency includes pancreatic enzyme replacement of lipase, protease, and amylase (e.g.,Pancreaze, Creon, Ultresa, Zenpep) administered before each meal and snack. This patient is at risk for hyponatremia based on reported symptoms. Adequate intake of fat, calories, protein, and vitamins is important. Fat-soluble vitamins (vitamins A, D, E, and K) must be supplemented because they are malabsorbed. Use of caloric supplements improves nutritional status. Hyperglycemia caused by pancreatic insufficiency is more likely to occur than hypoglycemia.

A patient arrives in the ear, nose, and throat clinic complaining of a piece of tissue being "stuck up my nose" and with foul-smelling nasal drainage from the right nare. Which action should the nurse take first? a. Notify the clinic health care provider. b. Obtain aerobic culture specimens of the drainage. c. Ask the patient about how the cotton got into the nose. d. Have the patient occlude the left nare and blow the nose.

have the patient occlude the left nare and blow the nose Rationale: Because the highest priority action is to remove the foreign object from the nare, the nurse's first action should be to assist the patient to remove the object. The other actions are also appropriate but should be done after attempting to clear the nose.

A 55-yr-old patient with increasing dyspnea is being evaluated for a possible diagnosis of chronic obstructive pulmonary disease (COPD). When teaching a patient about pulmonary spirometry for this condition, what is the most important question the nurse should ask? a. "Are you claustrophobic?" b. "Are you allergic to shellfish?" c. "Have you taken any bronchodilators today?" d. "Do you have any metal implants or prostheses?"

have you taken any bronchdilators today Rationale: Spirometry will help establish the COPD diagnosis. Bronchodilators should be avoided at least 6 hours before the test. Spirometry does not involve being placed in an enclosed area such as for magnetic resonance imaging (MRI). Contrast dye is not used for spirometry. The patient may still have spirometry done if metal implants or prostheses are present because they are contraindications for an MRI.

A nurse obtains a health history from a patient who has a 35 pack-year smoking history. The patient complains of hoarseness and tightness in the throat and difficulty swallowing. Which question is important for the nurse to ask? a. "How much alcohol do you drink in an average week?" b. "Do you have a family history of head or neck cancer?" c. "Have you had frequent streptococcal throat infections?" d. "Do you use antihistamines for upper airway congestion?"

how much alcohol do you drink in an average week Rationale: prolonged alcohol use and smoking are associated with the development of laryngeal cancer, which the patient's symptoms and history suggest. Family history is not a risk factor for head or neck cancer. Frequent antihistamine use would be asked about if the nurse suspected allergic rhinitis, but the patient's symptoms are not suggestive of this diagnosis. patients with streptococcal throat infections will also have pain and a fever.

the nurse plans to teach a patient how to manage allergic rhinitis. which information should the nurse include in the teaching plan? a. using oral antihistamines for 2 weeks before the allergy season may prevent reactions. b. identifying and avoiding environmental triggers are the best way to prevent symptoms c. frequent hand washing is the primary way to prevent spreading the condition to others d. corticosteroid nasal sprays will reduce inflammation, but systemic effects limit their use

identifying and avoiding environmental triggers are the best way to prevent symptoms Rationale: the most important intervention is to assist the patient in identifying and avoiding potential allergens. intranasal corticosteroids (not oral antihistamines) should be started several weeks before the allergy season. corticosteroid nasal sprays have minimal systemic absorption. acute viral rhinitis (common cold) can be prevented by washing hands, but allergic rhinitis cannot.

A patient with pneumonia has a fever of 101.4° F (38.6° C), a nonproductive cough, and an O2 saturation of 88%. The patient complains of weakness, fatigue, and needs assistance to get out of bed. Which nursing diagnosis should the nurse assign as the priority? a. Hyperthermia related to infectious illness b. Impaired transfer ability related to weakness c. Ineffective airway clearance related to thick secretions d. Impaired gas exchange related to respiratory congestion

impaired gas exchange related to respiratory congestion Rationale: All of these nursing diagnoses are appropriate for the patient, but the patient's O2 saturation indicates that all body tissues are at risk for hypoxia unless the gas exchange is improved.

The nurse assesses the chest of a patient with pneumococcal pneumonia. Which finding would the nurse expect? a. Increased tactile fremitus b. Dry, nonproductive cough c. Hyperresonance to percussion d. A grating sound on auscultation

increased tactile fremitus Rationale: Increased tactile fremitus over the area of pulmonary consolidation is expected with bacterial pneumonias. Dullness to percussion would be expected. Pneumococcal pneumonia typically presents with a loose, productive cough. Adventitious breath sounds such as crackles and wheezes are typical. A grating sound is more representative of a pleural friction rub rather than pneumonia.

On auscultation of a patient's lungs, the nurse hears low-pitched, bubbling sounds during inhalation in the lower third of both lungs. How should the nurse document this finding? a. Inspiratory crackles at the bases b. Expiratory wheezes in both lungs c. Abnormal lung sounds in the apices of both lungs d. Pleural friction rub in the right and left lower lobes

inspiratory crackles at the bases Rationale: Crackles are low-pitched, bubbling sounds usually heard on inspiration. Wheezes are high-pitched sounds. They can be heard during the expiratory or inspiratory phase of the respiratory cycle. The lower third of both lungs are the bases, not apices. Pleural friction rubs are grating sounds that are usually heard during both inspiration and expiration.

A patient with a chronic cough is scheduled to have a bronchoscopy with biopsy. Which intervention will the nurse implement directly after the procedure? a. Encourage the patient to drink clear liquids. b. Place the patient on bed rest for at least 4 hours. c. Keep the patient NPO until the gag reflex returns. d. Maintain the head of the bed elevated 90 degrees.

keep the patient NPO until the gas reflex returns Rationale: Risk for aspiration and maintaining an open airway is the priority. Because a local anesthetic is used to suppress the gag and cough reflexes during bronchoscopy, the nurse should monitor for the return of these reflexes before allowing the patient to take oral fluids or food. The patient does not need to be on bed rest, and the head of the bed does not need to be in the high-Fowler's position.

A nurse is caring for a patient who has had a total laryngectomy and radical neck dissection. During the first 24 hours after surgery what is the priority nursing action? a. Monitor the incision for bleeding. b. Maintain adequate IV fluid intake. c. Keep the patient in semi-Fowler's position. d. Teach the patient to suction the tracheostomy.

keep the patient in semi-Fowler's position Rationale: The most important goals after a laryngectomy and radical neck dissection are to maintain the airway and ensure adequate oxygenation. Keeping the patient in a semi-Fowler's position will decrease edema and limit tension on the suture lines to help ensure an open airway. Maintenance of IV fluids and monitoring for bleeding are important, but maintaining an open airway is the priority. During the immediate postoperative period, the patient with a laryngectomy requires frequent suctioning of the tracheostomy tube. The patient may be taught to suction after the tracheostomy is stable, if needed, but not during the immediate postoperative period.

Which action is appropriate for the nurse to delegate to unlicensed assistive personnel (UAP)? a. Listen to a patient's lung sounds for wheezes or crackles. b. Label specimens obtained during percutaneous lung biopsy. c. Instruct a patient about how to use home spirometry testing. d. Measure induration at the site of a patient's intradermal skin test

label specimens obtained during percutaneous lung biopsy Rationale: Labeling of specimens is within the scope of practice of UAP. The other actions require nursing judgment and should be done by licensed nursing personnel.

The nurse is caring for a patient with idiopathic pulmonary arterial hypertension (IPAH). Which assessment information requires the most immediate action by the nurse? a. The O2 saturation is 90%. b. The blood pressure is 98/56 mm Hg. c. The epoprostenol (Flolan) infusion is disconnected. d. The international normalized ratio (INR) is prolonged.

the epoprostenol (flolan) infusion is disconnected Rationale: The half-life of this drug is 6 minutes, so the nurse will need to restart the infusion as soon as possible to prevent rapid clinical deterioration. The other data also indicate a need for ongoing monitoring or intervention, but the priority action is to reconnect the infusion.

The nurse is admitting a patient diagnosed with an acute exacerbation of chronic obstructive pulmonary disease (COPD). How should the nurse determine the appropriate O2 flow rate? a. Minimize O2 use to avoid O2 dependency. b. Maintain the pulse oximetry level at 90% or greater. c. Administer O2 according to the patient's level of dyspnea. d. Avoid administration of O2 at a rate of more than 2 L/min

maintain the pulse oximetry at 90% or greater Rationale: The best way to determine the appropriate O2 flow rate is by monitoring the patient'soxygenation either by arterial blood gases (ABGs) or pulse oximetry. An O2 saturation of 90%indicates adequate blood O2 level without the danger of suppressing the respiratory drive. Forpatients with an exacerbation of COPD, an O2 flow rate of 2 L/min may not be adequate.Because O2 use improves survival rate in patients with COPD, there is no concern about O2dependency. The patient's perceived dyspnea level may be affected by other factors (e.g.,anxiety) besides blood O2 level.

The nurse notes that a patient has incisional pain, a poor cough effort, and scattered coarse crackles after a thoracotomy. Which action should the nurse take first? a. Assist the patient to sit upright in a chair. b. Splint the patient's chest during coughing. c. Medicate the patient with prescribed morphine. d. Observe the patient use the incentive spirometer

medicate the patient with prescribed morphine Rationale: A major reason for atelectasis and poor airway clearance in patients after chest surgery is incisional pain (which increases with deep breathing and coughing). The first action by the nurse should be to medicate the patient to minimize incisional pain. The other actions are all appropriate ways to improve airway clearance, but should be done after the morphine is given.

The nurse teaches a patient about the transmission of pulmonary tuberculosis (TB). Which statement, if made by the patient, indicates that teaching was effective? a. "I will take the bus instead of driving." b. "I will stay indoors whenever possible." c. "My spouse will sleep in another room." d. "I will keep the windows closed at home."

my spouse will sleep in another room Rationale: Teach the patient how to minimize exposure to close contacts and household members. Homes should be well ventilated, especially the areas where the infected person spends a lot of time. While still infectious, the patient should sleep alone, spend as much time as possible outdoors, and minimize time in congregate settings or on public transportation.

The nurse administers prescribed therapies for a patient with cor pulmonale and right-sided heart failure. Which assessment could be used to evaluate the effectiveness of the therapies? a. Observe for distended neck veins. b. Auscultate for crackles in the lungs. c. Palpate for heaves or thrills over the heart. d. Monitor for elevated white blood cell count.

observe for distended neck veins Rationale: Cor pulmonale is right ventricular failure caused by pulmonary hypertension, so clinical manifestations of right ventricular failure such as peripheral edema, jugular venous distention,and right upper-quadrant abdominal tenderness would be expected. Crackles in the lungs are likely to be heard with left-sided heart failure. Findings in cor pulmonale include evidence of right ventricular hypertrophy on electrocardiography and an increase in intensity of the second heart sound. Heaves or thrills are not common with cor pulmonale. White blood count elevation might indicate infection but is not expected with cor pulmonale.

A patient with chronic obstructive pulmonary disease (COPD) has a nursing diagnosis of imbalanced nutrition: less than body requirements. Which intervention would be most appropriate for the nurse to include in the plan of care? a. Encourage increased intake of whole grains. b. Increase the patient's intake of fruits and fruit juices. c. Offer high-calorie protein snacks between meals and at bedtime. d. Assist the patient in choosing foods with high vegetable content.

offer high-calorie protein snacks between meals and at bedtime Rationale: Eating small amounts more frequently (as occurs with snacking) will increase caloric intake by decreasing the fatigue and feelings of fullness associated with large meals. Patients with COPD should rest before meals. Foods that have a lot of texture such as whole grains may take more energy to eat and get absorbed and lead to decreased intake. Although fruits, juices,and minerals are not contraindicated, foods high in protein are a better choice

The nurse in the emergency department receives arterial blood gas results for four recently admitted patients with obstructive pulmonary disease. The results for which patient will require the most rapid action by the nurse? a. pH 7.28, PaCO2 50 mm Hg, and PaO2 58 mm Hg b. pH 7.48, PaCO2 30 mm Hg, and PaO2 65 mm Hg c. pH 7.34, PaCO2 33 mm Hg, and PaO2 80 mm Hg d. pH 7.31, PaCO2 58 mm Hg, and PaO2 64 mm Hg

pH 7.28, PaCO2 50 mm Hg, and PaO2 58 mm Hg Rationale: The pH, PaCO2, and PaO2 indicate that the patient has severe uncompensated respiratory acidosis and hypoxemia. Rapid action will be required to prevent increasing hypoxemia and correct the acidosis

The laboratory has just called with the arterial blood gas (ABG) results on four patients. Which result is most important for the nurse to report immediately to the health care provider? a. pH 7.34, PaO2 82 mm Hg, PaCO2 40 mm Hg, and O2 sat 97% b. pH 7.35, PaO2 85 mm Hg, PaCO2 50 mm Hg, and O2 sat 95% c. pH 7.46, PaO2 90 mm Hg, PaCO2 32 mm Hg, and O2 sat 98% d. pH 7.31, PaO2 91 mm Hg, PaCO2 50 mm Hg, and O2 sat 96%

pH 7.31, PaO2 01 mm Hg, PaCO2 50 mm Hg, and O2 sat 96% Rationale: These ABGs indicate uncompensated respiratory acidosis and should be reported to the health care provider. The other values are normal, close to normal, or compensated.

A patient experiences a chest wall contusion as a result of being struck in the chest with a baseball bat. The emergency department nurse would be most concerned if which finding is observed during the initial assessment? a. Paradoxical chest movement b. Complaint of chest wall pain c. Heart rate of 110 beats/minute d. Large bruised area on the chest

paradoxical chest movement Rationale: Paradoxical chest movement indicates that the patient may have flail chest, which can severely compromise gas exchange and can rapidly lead to hypoxemia. Chest wall pain, a slightly elevated pulse rate, and chest bruising all require further assessment or intervention,but the priority concern is poor gas exchange.

A patient with idiopathic pulmonary arterial hypertension (IPAH) is receiving nifedipine (Procardia). Which assessment would best indicate to the nurse that the patient's condition is improving? a. Patient's chest x-ray indicates clear lung fields. b. Heart rate is between 60 and 100 beats/minute. c. Patient reports a decrease in exertional dyspnea. d. Blood pressure (BP) is less than 140/90 mm Hg.

patient reports a decrease in exertional dyspnea Rationale: Because a major symptom of IPAH is exertional dyspnea, an improvement in this symptom would indicate that the medication was effective. Nifedipine will affect BP and heart rate, but these parameters would not be used to monitor the effectiveness of therapy for a patient with IPAH. The chest x-ray will show clear lung fields even if the therapy is not effective.

A young adult patient with cystic fibrosis (CF) is admitted to the hospital with increased dyspnea. Which intervention should the nurse include in the plan of care? a. Schedule a sweat chloride test. b. Arrange for a hospice nurse visit. c. Place the patient on a low-sodium diet. d. Perform chest physiotherapy every 4 hours

perform chest physiotherapy every 4 hours Rationale: Routine scheduling of airway clearance techniques is an essential intervention for patients with CF. A sweat chloride test is used to diagnose CF, but it does not provide any information about the effectiveness of therapy. There is no indication that the patient is terminally ill.Patients with CF lose excessive sodium in their sweat and require high amounts of dietary sodium

The nurse is caring for a patient with cor pulmonale. The nurse should monitor the patient for which expected finding? a. Chest pain b. Finger clubbing c. Peripheral edema d. Elevated temperature

peripheral edema Rationale: Cor pulmonale causes clinical manifestations of right ventricular failure, such as peripheral edema. The other clinical manifestations may occur in the patient with other complications of chronic obstructive pulmonary disease but are not indicators of cor pulmonale

A patient who has just been admitted with community-acquired pneumococcal pneumonia has a temperature of 101.6° F with a frequent cough and is complaining of severe pleuritic chest pain. Which prescribed medication should the nurse give first? a. Codeine b. Guaifenesin c. Acetaminophen (Tylenol) d. Piperacillin/tazobactam (Zosyn)

piperacillin/tazobactam (zosyn) Rationale: Early initiation of antibiotic therapy has been demonstrated to reduce mortality. The other medications are also appropriate and should be given as soon as possible, but the priority is to start antibiotic therapy.

Which action should the nurse plan to prevent aspiration in a high-risk patient? a. Turn and reposition an immobile patient at least every 2 hours. b. Place a patient with altered consciousness in a side-lying position. c. Insert a nasogastric tube for feeding a patient with high calorie needs. d. Monitor respiratory symptoms in a patient who is immunosuppressed.

place a patient with altered consciousness in a side-lying position Rationale: With loss of consciousness, the gag and cough reflexes are depressed, and aspiration is more likely to occur. The risk for aspiration is decreased when patients with a decreased level of consciousness are placed in a side-lying or upright position. Frequent turning prevents pooling of secretions in immobilized patients but will not decrease the risk for aspiration in patients at risk. Monitoring of parameters such as breath sounds and O2 saturation will help detect pneumonia in immunocompromised patients, but it will not decrease the risk for aspiration. Conditions that increase the risk of aspiration include decreased level of consciousness (e.g., seizure, anesthesia, head injury, stroke, alcohol intake), difficulty swallowing, and nasogastric intubation with or without tube feeding.

A patient in metabolic alkalosis is admitted to the emergency department and pulse oximetry (SpO2) indicates that the O2 saturation is 94%. Which action should the nurse expect to take next? a. Complete a head-to-toe assessment. b. Administer an inhaled bronchodilator. c. Place the patient on high-flow oxygen. d. Obtain repeat arterial blood gases (ABGs).

place the patient on high-flow oxygen Rationale: Although the O2 saturation is adequate, the left shift in the oxyhemoglobin dissociation curve will decrease the amount of O2 delivered to tissues, so high oxygen concentrations should be given. A head-to-toe assessment and repeat ABGs may be implemented later. Bronchodilators are not needed for metabolic alkalosis and there is no indication that the patient is having difficulty with airflow.

An experienced nurse instructs a new nurse about how to care for a patient with dyspnea caused by a pulmonary fungal infection. Which action by the new nurse indicates a need for further teaching? a. Listening to the patient's lung sounds several times during the shift b. Placing the patient on droplet precautions in a private hospital room c. Monitoring patient serology results to identify the infecting organism d. Increasing the O2 flow rate to keep the O2 saturation over 90%

placing the patient on droplet precautions in a private hospital room Rationale: Fungal infections are not transmitted from person to person. Therefore no isolation procedures are necessary. The other actions by the new nurse are appropriate.

A patient with a pleural effusion is scheduled for a thoracentesis. Which action should the nurse take to prepare the patient for the procedure? a. Start a peripheral IV line to administer sedatives. b. Position the patient sitting up on the side of the bed. c. Obtain a collection device to hold 3 liters of pleural fluid. d. Remind the patient not to eat or drink anything for 6 hours.

position the patient sitting up on the side of the bed Rationale: When the patient is sitting up, fluid accumulates in the pleural space at the lung bases and can more easily be located and removed. The patient does not usually require sedation for the procedure, and there are no restrictions on oral intake because the patient is not sedated or unconscious. Usually only 1000 to 1200 mL of pleural fluid is removed at one time. Rapid removal of a large volume can result in hypotension, hypoxemia, or pulmonary edema.

Which finding by the nurse for a patient with a nursing diagnosis of impaired gas exchange will be most useful in evaluating the effectiveness of treatment? a. Even, unlabored respirations b. Pulse oximetry reading of 92% c. Absence of wheezes or crackles d. Respiratory rate of 18 breaths/min

pulse oximetry reading of 92% Rationale: For the nursing diagnosis of impaired gas exchange, the best data for evaluation are arterial blood gases (ABGs) or pulse oximetry. The other data may indicate either improvement or impending respiratory failure caused by fatigue.

the nurse has just auscultated coarse crackles bilaterally on a patient with a tracheostomy tube in place. if the patient is unsuccessful in coughing up secretions, what action should the nurse take? a. encourage increased incentive spirometer use b. encourage the patient to increase oral fluid intake c. put on sterile gloves and use a sterile catheter to suction d. preoxygenate the patient for 3 minutes before suctioning

put on sterile gloves and use a sterile catheter to suction Rationale: This patient needs suctioning now to secure a patent airway. Sterile gloves and a sterile catheter are used when suctioning a tracheostomy. Preoxygenation for 3 minutes is not necessary; 30 seconds is recommended. Incentive spirometer use opens alveoli and can induce coughing, which can mobilize secretions. However, the patient with a tracheostomy may not be able to use an incentive spirometer. Increasing oral fluid intake would not moisten and helpmobilize secretions in a timely manner.

Using the illustrated technique, the nurse is assessing for which finding in a patient with chronic obstructive pulmonary disease (COPD)? (palpation) a. Hyperresonance b. Tripod positioning c. Reduced excursion d. Accessory muscle use

reduced excursion Rationale: The technique for palpation for chest excursion is shown in the illustrated technique. Reduced chest movement would be noted on palpation of a patient's chest with COPD. Hyperresonance would be assessed through percussion. Accessory muscle use and tripod positioning would be assessed by inspection

An occupational health nurse works at a manufacturing plant where there is potential exposure to inhaled dust. Which action recommended by the nurse is intended to prevent lung disease? a. Treat workers with pulmonary fibrosis. b. Teach about symptoms of lung disease. c. Require the use of protective equipment. d. Monitor workers for coughing and wheezing.

require the use of protective equipment Rationale: Prevention of lung disease requires the use of appropriate protective equipment such as masks. The other actions will help in recognition or early treatment of lung disease but will not be effective in prevention of lung damage. Repeated exposure eventually results in diffuse pulmonary fibrosis. Fibrosis is the result of tissue repair after inflammation.

Which information about prevention of lung disease should the nurse include for a patient with a 42 pack-year history of cigarette smoking? a. Resources for support in smoking cessation b. Reasons for annual sputum cytology testing c. Erlotinib (Tarceva) therapy to prevent tumor risk d. Computed tomography (CT) screening for cancer

resources for support in smoking cessation Rationale: Because smoking is the major cause of lung cancer, the most important role for the nurse is teaching patients about the benefits of and means of smoking cessation. CT scanning is currently being investigated as a screening test for high-risk patients. However, if there is a positive finding, the person already has lung cancer. Sputum cytology is a diagnostic test, but does not prevent cancer or disease. Erlotinib may be used in patients who have lung cancer,but it is not used to reduce the risk of developing cancer.

The nurse assesses a patient with chronic obstructive pulmonary disease (COPD) who has been admitted with increasing dyspnea over the past 3 days. Which finding is important for the nurse to report to the health care provider? a. Respirations are 36 breaths/min. b. Anterior-posterior chest ratio is 1:1. c. Lung expansion is decreased bilaterally. d. Hyperresonance to percussion is present

respirations are 36 breaths/min Rationale: The increase in respiratory rate indicates respiratory distress and a need for rapid interventions such as administration of O2 or medications. The other findings are common chronic changes occurring in patients with COPD.

The nurse prepares a patient with a left-sided pleural effusion for a thoracentesis. How should the nurse position the patient? a. High-Fowler's position with the left arm extended b. Supine with the head of the bed elevated 30 degrees c. On the right side with the left arm extended above the head d. Sitting upright with the arms supported on an over bed table

sitting upright with the arms supported on an over bed table Rationale: The upright position with the arms supported increases lung expansion, allows fluid to collect at the lung bases, and expands the intercostal space so that access to the pleural space is easier. The other positions would increase the work of breathing for the patient and make it more difficult for the health care provider performing the thoracentesis.

A patient is admitted with active tuberculosis (TB). The nurse should question a health care provider's order to discontinue airborne precautions unless which assessment finding is documented? a. Chest x-ray shows no upper lobe infiltrates. b. TB medications have been taken for 6 months. c. Mantoux testing shows an induration of 10 mm. d. Sputum smears for acid-fast bacilli are negative.

sputum smears for acid-fast bacilli are negative Rationale: Repeated negative sputum smears indicate that Mycobacterium tuberculosis is not present in the sputum, and the patient cannot transmit the bacteria by the airborne route. Chest x-rays are not used to determine whether treatment has been successful. Taking medications for 6 months is necessary, but the multidrug-resistant forms of the disease might not be eradicated after 6 months of therapy. Repeat Mantoux testing would not be done because the result will not change even with effective treatment.

A patient admitted to the emergency department complaining of sudden onset shortness of breath is diagnosed with a possible pulmonary embolus. How should the nurse prepare the patient for diagnostic testing to confirm the diagnosis? a. Ensure that the patient has been NPO. b. Start an IV so contrast media may be given. c. Inform radiology that radioactive glucose preparation is needed. d. Instruct the patient to expect to inspire deeply and exhale forcefully.

start an IV so contrast media may be given Rationale: Spiral computed tomography scans are the most commonly used test to diagnose pulmonary emboli and contrast media may be given IV. Bronchoscopy is used to detect changes in the bronchial tree, not to assess for vascular changes, and the patient should be NPO 6 to 12 hours before the procedure. Positron emission tomography scans are most useful in determining the presence of malignancy and a radioactive glucose preparation is used. For spirometry, the patient is asked to inhale deeply and exhale as long, hard, and fast as possible.

Which nursing action could the registered nurse (RN) working in a skilled care hospital unit delegate to an experienced licensed practical/vocational nurse (LPN/LVN) caring for a patient with a permanent tracheostomy? a. Assess the patient's risk for aspiration. b. Suction the tracheostomy when directed. c. Teach the patient to provide tracheostomy self-care. d. Determine the need for tracheostomy tube replacement.

suction the tracheostomy when directed Rationale: Suctioning of a stable patient can be delegated to LPNs/LVNs. Patient assessment and patient teaching should be done by the RN.

A patient is admitted to the emergency department with an open stab wound to the left chest. What action should the nurse take? a. Keep the head of the patient's bed positioned flat. b. Cover the wound tightly with an occlusive dressing. c. Position the patient so that the left chest is dependent. d. Tape a nonporous dressing on three sides over the wound.

tape a nonporous dressing on three sides over the wound Rationale: The dressing taped on three sides will allow air to escape when intrapleural pressure increases during expiration, but it will prevent air from moving into the pleural space during inspiration.Placing the patient on the left side or covering the chest wound with an occlusive dressing will allow trapped air in the pleural space and cause tension pneumothorax. The head of the bed should be elevated to 30 to 45 degrees to facilitate breathing.

Which intervention will the nurse include in the plan of care for a patient who is diagnosed with a lung abscess? a. Teach the patient to avoid the use of over-the-counter expectorants. b. Assist the patient with chest physiotherapy and postural drainage. c. Notify the health care provider immediately about any bloody or foul-smelling sputum. d. Teach about the need for prolonged antibiotic therapy after discharge from the hospital.

teach about the need for prolonged antibiotic therapy after discharge from the hospital Rationale: Long-term antibiotic therapy is needed for effective eradication of the infecting organisms in lung abscess. Chest physiotherapy and postural drainage are not recommended for lung abscess because they may lead to spread of the infection. Foul-smelling and bloody sputum are common clinical manifestations in lung abscess. Expectorants may be used because the patient is encouraged to cough.

The nurse is caring for a patient who has acute pharyngitis caused by Candida albicans. Which action is appropriate for the nurse to include in the plan of care? a. Assess patient for allergies to penicillin antibiotics. b. Teach the patient to sleep in a warm, dry environment. c. Avoid giving the patient warm food or warm liquids to drink. d. Teach patient to "swish and swallow" prescribed oral nystatin

teach patient to "swish and swallow" prescribed oral nystatin Rationale: Oral or pharyngeal fungal infections are treated with nystatin solution. The goal of the "swish and swallow" technique is to expose all of the oral mucosa to the antifungal agent. Warm liquids may be soothing to a sore throat. The patient should be taught to use a cool mist humidifier. There is no need to assess for penicillin and cephalosporin allergies because Candida albicans infection is treated with antifungals

The health care provider writes an order for bacteriologic testing for a patient who has a positive tuberculosis skin test. Which action should the nurse take? a. Teach about the reason for the blood tests. b. Schedule an appointment for a chest x-ray. c. Teach the patient about providing specimens for 3 consecutive days. d. Instruct the patient to collect several separate sputum specimens today.

teach the patient about providing specimens for 3 consecutive days rationale: Sputum specimens are obtained on 2 to 3 consecutive days for bacteriologic testing for Mycobacterium tuberculosis. The patient should not provide all the specimens at once. Blood cultures are not used for tuberculosis testing. A chest x-ray is not bacteriologic testing. Although the findings on chest x-ray examination are important, it is not possible to make a diagnosis of TB solely based on chest x-ray findings because other diseases can mimic the appearance of TB

The home health nurse is visiting a patient with chronic obstructive pulmonary disease (COPD). Which nursing action is appropriate to implement for a nursing diagnosis of impaired breathing pattern related to anxiety? a. Titrate O2 to keep saturation at least 90%. b. Teach the patient how to use pursed-lip breathing. c. Discuss a high-protein, high-calorie diet with the patient. d. Suggest the use of over-the-counter sedative medications.

teach the patient how to use pursed-lip breathing Rationale: Pursed-lip breathing techniques assist in prolonging the expiratory phase of respiration and decrease air trapping. There is no indication that the patient requires O2 therapy or an improved diet. Sedative medications should be avoided because they decrease respiratory drive

Which action by the nurse will be most effective in decreasing the spread of pertussis in a community setting? a. Providing supportive care to patients diagnosed with pertussis b. Teaching family members about the need for careful hand washing c. Teaching patients about the need for adult pertussis immunizations d. Encouraging patients to complete the prescribed course of antibiotics

teaching patients about the need for adult pertussis immunizations Rationale: The increased rate of pertussis in adults is thought to be caused by decreasing immunity after childhood immunization. Immunization is the most effective method of protecting communities from infectious diseases. Hand washing should be taught, but pertussis is spread by droplets and contact with secretions. Supportive care does not shorten the course of the disease or the risk for transmission. Taking antibiotics as prescribed does assist with decreased transmission, but patients are likely to have already transmitted the disease by the time the diagnosis is made.

A lobectomy is scheduled for a patient with stage I non-small cell lung cancer. The patient tells the nurse, "I would rather have chemotherapy than surgery." Which response by the nurse is most appropriate? a. "Are you afraid that the surgery will be very painful?" b. "Did you have bad experiences with previous surgeries?" c. "Tell me what you know about the treatments available." d. "Surgery is the treatment of choice for stage I lung cancer."

tell me what you know about the treatments available Rationale: More assessment of the patient's concerns about surgery is indicated. An open-ended response will elicit the most information from the patient. The answer beginning, "Surgery is the treatment of choice" is accurate, but it discourages the patient from sharing concerns about surgery. The remaining two answers indicate that the nurse has jumped to conclusions about the patient's reasons for not wanting surgery. Chemotherapy is the primary treatment for small cell lung cancer. In non-small cell lung cancer, chemotherapy may be used in the treatment of nonresectable tumors or as adjuvant therapy to surgery.

The nurse is caring for a hospitalized older patient who has nasal packing in place after a nosebleed. Which assessment finding will require the most immediate action by the nurse? a. The oxygen saturation is 89%. b. The nose appears red and swollen. c. The patient reports level 8 (0 to 10 scale) pain. d. The patient's temperature is 100.1° F (37.8° C).

the oxygen saturation is 89% Rationale: Older patients with nasal packing are at risk of aspiration or airway obstruction. An O2 saturation of 89% should alert the nurse to further assess for these complications. The other assessment data also indicate a need for nursing action but not as immediately as the low O2 saturation

The nurse analyzes the results of a patient's arterial blood gases (ABGs). Which finding would require immediate action? a. The bicarbonate level (HCO3-) is 31 mEq/L. b. The arterial oxygen saturation (SaO2) is 92%. c. The partial pressure of CO2 in arterial blood (PaCO2) is 31 mm Hg. d. The partial pressure of oxygen in arterial blood (PaO2) is 59 mm Hg.

the partial pressure of oxygen in arterial blood (PaO2) is 59 mm Hg Rationale: All the values are abnormal, but the low PaO2 indicates that the patient is at the point on the oxyhemoglobin dissociation curve where a small change in the PaO2 will cause a large drop in the O2 saturation and a decrease in tissue oxygenation. The nurse should intervene immediately to improve the patient's oxygenation.

A patient who had a total laryngectomy has a nursing diagnosis of hopelessness related to loss of control of personal care. Which information obtained by the nurse indicates that this identified problem is resolving? a. The patient allows the nurse to suction the tracheostomy. b. The patient's spouse provides the daily tracheostomy care. c. The patient asks how to clean the tracheostomy stoma and tube. d. The patient uses a communication board to request "No Visitors."

the patient asks how to clean the tracheostomy stoma and tube Rationale: Independently caring for the laryngectomy tube indicates that the patient has regained controlof personal care and hopelessness is at least partially resolved. Letting the nurse and spouseprovide care and requesting no visitors may indicate that the patient is still experiencinghopelessness.

A patient is diagnosed with both human immunodeficiency virus (HIV) and active tuberculosis (TB) disease. Which information obtained by the nurse is most important to communicate to the health care provider? a. The Mantoux test had an induration of 7 mm. b. The chest-x-ray showed infiltrates in the lower lobes. c. The patient has a cough that is productive of blood-tinged mucus. d. The patient is being treated with antiretrovirals for HIV infection.

the patient is being treated with antiretrovirals for HIV infection Rationale: Drug interactions can occur between the antiretrovirals used to treat HIV infection and the medications used to treat TB. The other data are expected in a patient with HIV and TB.

The nurse teaches a patient about pursed-lip breathing. Which action by the patient would indicate to the nurse that further teaching is needed? a. The patient inhales slowly through the nose. b. The patient puffs up the cheeks while exhaling. c. The patient practices by blowing through a straw. d. The patient's ratio of inhalation to exhalation is 1:3

the patient puffs up the cheeks while exhaling Rationale: The patient should relax the facial muscles without puffing the cheeks while doing pursed-lip breathing. The other actions by the patient indicate a good understanding of pursed-lip breathing.

The nurse teaches a patient with chronic bronchitis about a new prescription for Advair Diskus (combined fluticasone and salmeterol). Which action by the patient would indicate to the nurse that teaching about medication administration has been successful? a. The patient shakes the device before use. b. The patient rapidly inhales the medication. c. The patient attaches a spacer to the Diskus. d. The patient performs huff coughing after inhalation.

the patient rapidly inhales the medication Rationale: The patient should inhale the medication rapidly. Otherwise the dry particles will stick to the tongue and oral mucosa and not get inhaled into the lungs. Advair Diskus is a dry powder inhaler; shaking is not recommended. Spacers are not used with dry powder inhalers. Huff coughing is a technique to move mucus into larger airways to expectorate. The patient should not huff cough or exhale forcefully after taking Advair in order to keep the medication in the lungs.

The nurse teaches a patient how to administer formoterol (Perforomist) through a nebulizer. Which action by the patient indicates good understanding of the teaching? a. The patient attaches a spacer before using the inhaler. b. The patient coughs vigorously after using the inhaler. c. The patient removes the facial mask when misting stops. d. The patient activates the inhaler at the onset of expiration.

the patient removes the facial mask when misting stops Rationale: A nebulizer is used to administer aerosolized medication. A mist is seen when the medication is aerosolized, and when all of the medication has been used, the misting stops. The other options refer to inhaler use. Coughing vigorously after inhaling and activating the inhaler at the onset of expiration are both incorrect techniques when using an inhaler

The nurse interviews a patient with a new diagnosis of chronic obstructive pulmonary disease (COPD). Which information is most specific in confirming a diagnosis of chronic bronchitis? a. The patient tells the nurse about a family history of bronchitis. b. The patient indicates a 30 pack-year cigarette smoking history. c. The patient reports a productive cough for 3 months every winter. d. The patient denies having respiratory problems until the past 12 months.

the patient reports a productive cough for 3 months every winter Rationale: A diagnosis of chronic bronchitis is based on a history of having a productive cough for 3 months for at least 2 consecutive years. There is no family tendency for chronic bronchitis.Although smoking is the major risk factor for chronic bronchitis, a smoking history does not confirm the diagnosis

The nurse is caring for a patient with chronic obstructive pulmonary disease (COPD). Which information obtained from the patient would prompt the nurse to consult with the health care provider before administering the prescribed theophylline? a. The patient reports a recent 15-lb weight gain. b. The patient denies shortness of breath at present. c. The patient takes cimetidine (Tagamet HB) daily. d. The patient complains of coughing up green mucus.

the patient takes cimetidine (tagamet HB) daily Rationale: Cimetidine interferes with the metabolism of theophylline, and concomitant administration may lead rapidly to theophylline toxicity. The other patient information would not affect whether the theophylline should be administered or not.

A patient with right lower-lobe pneumonia has been treated with IV antibiotics for 3 days. Which assessment data obtained by the nurse indicates that the treatment is effective? a. Bronchial breath sounds are heard at the right base. b. The patient coughs up small amounts of green mucus. c. The patient's white blood cell (WBC) count is 9000/µL. d. Increased tactile fremitus is palpable over the right chest.

the patient's white blood cell (WBC) count is 9000/µL Rationale: the normal WBC count indicates that the antibiotics have been effective. All the other data suggest that a change in treatment is needed.

The nurse observes a student who is listening to a patient's lungs. Which action by the student indicates a need to review respiratory assessment skills? a. The student compares breath sounds from side to side at each level. b. The student listens during the inspiratory phase, then moves the stethoscope. c. The student starts at the apices of the lungs, moving down toward the lung bases. d. The student instructs the patient to breathe slowly and deeply through the mouth.

the student listens during the inspiratory phase, then moves the stethoscope Rationale: Listening only during inspiration indicates the student needs a review of respiratory assessment skills. At each placement of the stethoscope, listen to at least one cycle of inspiration and expiration. During chest auscultation, instruct the patient to breathe slowly and a little deeper than normal through the mouth. Auscultation should proceed from the lung apices to the bases, comparing opposite areas of the chest, unless the patient is in respiratory distress or will tire easily.

Which information will the nurse include in the asthma teaching plan for a patient being discharged? a. Use the inhaled corticosteroid when shortness of breath occurs. b. Inhale slowly and deeply when using the dry powder inhaler (DPI). c. Hold your breath for 5 seconds after using the bronchodilator inhaler. d. Tremors are an expected side effect of rapidly acting bronchodilators.

tremors are an expected side effect of rapidly acting bronchodilators Rationale: Tremors are a common side effect of short-acting 2-adrenergic (SABA) medications and not a reason to avoid using the SABA inhaler. Inhaled corticosteroids do not act rapidly to reduce dyspnea. Rapid inhalation is needed when using a DPI. The patient should hold the breath for 10 seconds after using inhalers.

the nurse is caring for a mechanically ventilated patient with a cuffed tracheostomy tube. which action by the nurse would determine if the cuff has been properly inflated? a. use a hand-held manometer to measure cuff pressure b. review the health record for the prescribed cuff pressure c. suction the patient through a fenestrated inner cannula to clear secretions d. insert the decannulation plug before removing the nonfenestrated inner cannula

use a hand-held manometer to measure cuff pressure Rationale: Measurement of cuff pressure using a manometer to ensure that cuff pressure is 20 mm Hg or lower will avoid compression of the tracheal wall and capillaries. Never insert the decannulation plug in a tracheostomy tube until the cuff is deflated and the nonfenestrated inner cannula is removed. Otherwise, the patient's airway is occluded. A health care provider's order is not required to determine safe cuff pressure. A nonfenestrated inner cannula must be used to suction a patient to prevent tracheal damage occurring from the suction catheter passing through the fenestrated openings.

When assessing a patient with a sore throat, the nurse notes anterior cervical lymph node swelling, a temperature of 101.6° F (38.7° C), and yellow patches on the tonsils. Which action will the nurse anticipate taking? a. Teach the patient about the use of expectorants. b. Use a swab to obtain a sample for a rapid strep antigen test. c. Discuss the need to rinse the mouth out after using any inhalers. d. Teach the patient to avoid nonsteroidal antiinflammatory drugs (NSAIDs).

use a swab to obtain a sample for a rapid strep antigen test Rationale: The patient's clinical manifestations are consistent with streptococcal pharyngitis, and the nurse will anticipate the need for a rapid strep antigen test or cultures (or both). Because patients with streptococcal pharyngitis usually do not have a cough, use of expectorants will not be anticipated. Rinsing out the mouth after inhaler use may prevent fungal oral infections, but the patient's assessment data are not consistent with a fungal infection. NSAIDs are frequently prescribed for pain and fever relief with pharyngitis.

Employee health test results reveal a tuberculosis (TB) skin test of 16-mm induration and a negative chest x-ray for a staff nurse working on the pulmonary unit. The nurse has no symptoms of TB. Which information should the occupational health nurse plan to teach the staff nurse? a. Use and side effects of isoniazid b. Standard four-drug therapy for TB c. Need for annual repeat TB skin testing d. Bacille Calmette-Guérin (BCG) vaccine

use and side effects of isoniazid Rationale: The nurse is considered to have a latent TB infection and should be treated with INH daily for 6 to 9 months. The four-drug therapy would be appropriate if the nurse had active TB. TB skin testing is not done for individuals who have already had a positive skin test result. BCG vaccine is not used in the United States for TB and would not be helpful for this individual, who already has a TB infection.

After assessment of a patient with pneumonia, the nurse identifies a nursing diagnosis of ineffective airway clearance. Which assessment data best supports this diagnosis? a. Weak cough effort b. Profuse green sputum c. Respiratory rate of 28 breaths/minute d. Resting pulse oximetry (SpO2) of 85%

weak cough effort Rationale: The weak, nonproductive cough indicates that the patient is unable to clear the airway effectively. The other data would be used to support diagnoses such as impaired gas exchange and ineffective breathing pattern.

A patient is scheduled for spirometry. Which action should the nurse take to prepare the patient for this procedure? a. Give the rescue medication immediately before testing. b. Administer oral corticosteroids 2 hours before the procedure. c. Withhold bronchodilators for 6 to 12 hours before the examination. d. Ensure that the patient has been NPO for several hours before the test.

withhold bronchodilators for 6 to 12 hours before the procedure Rationale: Bronchodilators are held before spirometry so that a baseline assessment of airway function can be determined. Testing is repeated after bronchodilator use to determine whether the decrease in lung function is reversible. There is no need for the patient to be NPO. Oral corticosteroids should be held before spirometry. Rescue medications (which are bronchodilators) would not be given until after the baseline pulmonary function was assessed.

An older patient is receiving standard multidrug therapy for tuberculosis (TB). The nurse should notify the health care provider if the patient exhibits which finding? a. Yellow-tinged sclera b. Orange-colored sputum c. Thickening of the fingernails d. Difficulty hearing high-pitched voices

yellow-tinged sclera Rationale: Noninfectious hepatitis is a toxic effect of isoniazid, rifampin, and pyrazinamide, and patients who develop hepatotoxicity will need to use other medications. Changes in hearing and nail thickening are not expected with the four medications used for initial TB drug therapy. Presbycusis is an expected finding in the older adult patient. Orange discoloration of body fluids is an expected side effect of rifampin and not an indication to call the health care provider.

A patient scheduled for a total laryngectomy and radical neck dissection for cancer of the larynx asks the nurse, "Will I be able to talk normally after surgery?" What is the most accurate response by the nurse? a. "You will breathe through a permanent opening in your neck, but you will not be able to communicate orally." b. "You won't be able to talk right after surgery, but you will be able to speak again after the tracheostomy tube is removed." c. "You will have a permanent opening into your neck, and you will need rehabilitation for some type of voice restoration." d. "You won't be able to speak as you used to, but there are artificial voice devices that will give you the ability to speak normally."

you will have a permanent opening into your neck, and you will need rehabilitation for some type of voice restoration. Rationale: Voice rehabilitation is planned after a total laryngectomy, and a variety of assistive devices are available to restore communication. Although the ability to communicate orally is changed, it would not be lost. Artificial voice devices do not permit normal-sounding speech. In a total laryngectomy, the vocal cords are removed, so normal speech is impossible.


Conjuntos de estudio relacionados

Pediatrics Midterm Review Questions

View Set

Exam 1 (Endocrine System, Blood, Cardiovascular System)

View Set

Chapter 31 Drug Therapy Nasal Congestion and cough

View Set

Pharmacology ATI study questions part 4

View Set

Algebra 1 Math Final Study Guide

View Set

5.8 Financial Responsibility, Insurance, and Collisions Quiz

View Set

Ch 71 Cyclooxygenase Inhibitors: Nonsteroidal Anti-Inflammatory Drugs and Acetaminophen

View Set

Prepare: Worksheet 13.1: What is Consideration?

View Set